Correct answer to this modulo

The name of the pictureThe name of the pictureThe name of the pictureClash Royale CLAN TAG#URR8PPP











up vote
2
down vote

favorite












What is the correct answer to this expression:



$26^32 pmod 12$



When I tried in Wolfram Alpha the answer is $4$, this is also my answer using Fermat's little theorem, but in a calculator the answer is different, $0.$










share|cite|improve this question



















  • 3




    How can it be $0$? $3$ does not divide $26$, so $12 (= 3 times 4)$ can't divide $26^32$
    – ab123
    1 hour ago







  • 1




    So the answer 4 is correct?
    – Naruto Uzumaki
    1 hour ago







  • 1




    yes the answer $4$ is correct
    – ab123
    1 hour ago







  • 2




    I would suggest that, depending on exactly how you did this on a calculator, at some stage the calculator encountered a very large exponent and truncated the result, throwing off the rest of the calculation.
    – Sam Streeter
    1 hour ago










  • Alpha failing on such a "basic" computation is more than unlikely. Even the Windows calculator returns $4$ ! (by chance ?)
    – Yves Daoust
    5 mins ago














up vote
2
down vote

favorite












What is the correct answer to this expression:



$26^32 pmod 12$



When I tried in Wolfram Alpha the answer is $4$, this is also my answer using Fermat's little theorem, but in a calculator the answer is different, $0.$










share|cite|improve this question



















  • 3




    How can it be $0$? $3$ does not divide $26$, so $12 (= 3 times 4)$ can't divide $26^32$
    – ab123
    1 hour ago







  • 1




    So the answer 4 is correct?
    – Naruto Uzumaki
    1 hour ago







  • 1




    yes the answer $4$ is correct
    – ab123
    1 hour ago







  • 2




    I would suggest that, depending on exactly how you did this on a calculator, at some stage the calculator encountered a very large exponent and truncated the result, throwing off the rest of the calculation.
    – Sam Streeter
    1 hour ago










  • Alpha failing on such a "basic" computation is more than unlikely. Even the Windows calculator returns $4$ ! (by chance ?)
    – Yves Daoust
    5 mins ago












up vote
2
down vote

favorite









up vote
2
down vote

favorite











What is the correct answer to this expression:



$26^32 pmod 12$



When I tried in Wolfram Alpha the answer is $4$, this is also my answer using Fermat's little theorem, but in a calculator the answer is different, $0.$










share|cite|improve this question















What is the correct answer to this expression:



$26^32 pmod 12$



When I tried in Wolfram Alpha the answer is $4$, this is also my answer using Fermat's little theorem, but in a calculator the answer is different, $0.$







modular-arithmetic






share|cite|improve this question















share|cite|improve this question













share|cite|improve this question




share|cite|improve this question








edited 1 hour ago









amWhy

191k27223437




191k27223437










asked 1 hour ago









Naruto Uzumaki

185




185







  • 3




    How can it be $0$? $3$ does not divide $26$, so $12 (= 3 times 4)$ can't divide $26^32$
    – ab123
    1 hour ago







  • 1




    So the answer 4 is correct?
    – Naruto Uzumaki
    1 hour ago







  • 1




    yes the answer $4$ is correct
    – ab123
    1 hour ago







  • 2




    I would suggest that, depending on exactly how you did this on a calculator, at some stage the calculator encountered a very large exponent and truncated the result, throwing off the rest of the calculation.
    – Sam Streeter
    1 hour ago










  • Alpha failing on such a "basic" computation is more than unlikely. Even the Windows calculator returns $4$ ! (by chance ?)
    – Yves Daoust
    5 mins ago












  • 3




    How can it be $0$? $3$ does not divide $26$, so $12 (= 3 times 4)$ can't divide $26^32$
    – ab123
    1 hour ago







  • 1




    So the answer 4 is correct?
    – Naruto Uzumaki
    1 hour ago







  • 1




    yes the answer $4$ is correct
    – ab123
    1 hour ago







  • 2




    I would suggest that, depending on exactly how you did this on a calculator, at some stage the calculator encountered a very large exponent and truncated the result, throwing off the rest of the calculation.
    – Sam Streeter
    1 hour ago










  • Alpha failing on such a "basic" computation is more than unlikely. Even the Windows calculator returns $4$ ! (by chance ?)
    – Yves Daoust
    5 mins ago







3




3




How can it be $0$? $3$ does not divide $26$, so $12 (= 3 times 4)$ can't divide $26^32$
– ab123
1 hour ago





How can it be $0$? $3$ does not divide $26$, so $12 (= 3 times 4)$ can't divide $26^32$
– ab123
1 hour ago





1




1




So the answer 4 is correct?
– Naruto Uzumaki
1 hour ago





So the answer 4 is correct?
– Naruto Uzumaki
1 hour ago





1




1




yes the answer $4$ is correct
– ab123
1 hour ago





yes the answer $4$ is correct
– ab123
1 hour ago





2




2




I would suggest that, depending on exactly how you did this on a calculator, at some stage the calculator encountered a very large exponent and truncated the result, throwing off the rest of the calculation.
– Sam Streeter
1 hour ago




I would suggest that, depending on exactly how you did this on a calculator, at some stage the calculator encountered a very large exponent and truncated the result, throwing off the rest of the calculation.
– Sam Streeter
1 hour ago












Alpha failing on such a "basic" computation is more than unlikely. Even the Windows calculator returns $4$ ! (by chance ?)
– Yves Daoust
5 mins ago




Alpha failing on such a "basic" computation is more than unlikely. Even the Windows calculator returns $4$ ! (by chance ?)
– Yves Daoust
5 mins ago










4 Answers
4






active

oldest

votes

















up vote
2
down vote



accepted










First, note that $26 equiv 2 pmod 12$, so $26^32 equiv 2^32 pmod 12$.



Next, note that $2^4 equiv 16 equiv 4 pmod 12$, so $2^32 equiv left(2^4right)^8 equiv 4 ^8 pmod 12$, and $4^2 equiv 4 pmod 12$.



Finally $4^8 equiv left(4^2right)^4 equiv 4^4 equiv left(4^2right)^2 equiv 4^2 equiv 4 pmod 12$.



Then we get the result.



There are slicker solutions with just a few results from elementary number theory, but this is a very basic method which should be easy enough to follow.






share|cite|improve this answer



























    up vote
    1
    down vote













    Like



    Get the last two digits of $16^100$ and $17^100$



    How to find last two digits of $2^2016$



    last two digits of $14^5532$?



    Find the last two digits of $2^2156789$



    $$26equiv-1pmod3$$



    $implies26^2nequiv(-1)^2nequiv1pmod3$ where $n$ is any integer



    $implies26^2n+2equiv1cdot26^2pmod3cdot26^2$



    $implies26^2n+2equiv26^2pmod3cdot2^2equiv?$






    share|cite|improve this answer



























      up vote
      1
      down vote













      Note that $26equiv 2pmod12$, so we can as well compute $r=2^32bmod 12$.



      We have $2^32=12q+r$, so clearly $r=4s$, so we are reduced to compute $2^30bmod 3$ and now we can apply Fermat’s little theorem: $2^2equiv 1pmod 3$; thus $2^30equiv 1=spmod3$ and therefore $r=4$.






      share|cite|improve this answer



























        up vote
        0
        down vote













        Note that $26^32$ is clearly divisible by $4$ but not by $3$, so the possible congruence classes modulo $12$ are immediately only (represented by) $4$ and $8$. We have $4equiv 1 bmod 3$ and $8equiv 2 equiv -1 bmod 3$, so to distinguish between them we only need to know the answer modulo $3$.



        This comes down to $(-1)^32=1$ (or we can use little Fermat to the same effect) and the answer is therefore $4$.






        share|cite|improve this answer




















        • Splitting the $12$ into the coprime factors $4$ and $3$ is a technique which is substantially generalised in the Chinese Remainder Theorem. The observations in this case are elementary and don't need complex machinery, but it is there if needed to steer the analysis of more complex cases.
          – Mark Bennet
          14 mins ago










        Your Answer




        StackExchange.ifUsing("editor", function ()
        return StackExchange.using("mathjaxEditing", function ()
        StackExchange.MarkdownEditor.creationCallbacks.add(function (editor, postfix)
        StackExchange.mathjaxEditing.prepareWmdForMathJax(editor, postfix, [["$", "$"], ["\\(","\\)"]]);
        );
        );
        , "mathjax-editing");

        StackExchange.ready(function()
        var channelOptions =
        tags: "".split(" "),
        id: "69"
        ;
        initTagRenderer("".split(" "), "".split(" "), channelOptions);

        StackExchange.using("externalEditor", function()
        // Have to fire editor after snippets, if snippets enabled
        if (StackExchange.settings.snippets.snippetsEnabled)
        StackExchange.using("snippets", function()
        createEditor();
        );

        else
        createEditor();

        );

        function createEditor()
        StackExchange.prepareEditor(
        heartbeatType: 'answer',
        convertImagesToLinks: true,
        noModals: false,
        showLowRepImageUploadWarning: true,
        reputationToPostImages: 10,
        bindNavPrevention: true,
        postfix: "",
        noCode: true, onDemand: true,
        discardSelector: ".discard-answer"
        ,immediatelyShowMarkdownHelp:true
        );



        );













         

        draft saved


        draft discarded


















        StackExchange.ready(
        function ()
        StackExchange.openid.initPostLogin('.new-post-login', 'https%3a%2f%2fmath.stackexchange.com%2fquestions%2f2973194%2fcorrect-answer-to-this-modulo%23new-answer', 'question_page');

        );

        Post as a guest






























        4 Answers
        4






        active

        oldest

        votes








        4 Answers
        4






        active

        oldest

        votes









        active

        oldest

        votes






        active

        oldest

        votes








        up vote
        2
        down vote



        accepted










        First, note that $26 equiv 2 pmod 12$, so $26^32 equiv 2^32 pmod 12$.



        Next, note that $2^4 equiv 16 equiv 4 pmod 12$, so $2^32 equiv left(2^4right)^8 equiv 4 ^8 pmod 12$, and $4^2 equiv 4 pmod 12$.



        Finally $4^8 equiv left(4^2right)^4 equiv 4^4 equiv left(4^2right)^2 equiv 4^2 equiv 4 pmod 12$.



        Then we get the result.



        There are slicker solutions with just a few results from elementary number theory, but this is a very basic method which should be easy enough to follow.






        share|cite|improve this answer
























          up vote
          2
          down vote



          accepted










          First, note that $26 equiv 2 pmod 12$, so $26^32 equiv 2^32 pmod 12$.



          Next, note that $2^4 equiv 16 equiv 4 pmod 12$, so $2^32 equiv left(2^4right)^8 equiv 4 ^8 pmod 12$, and $4^2 equiv 4 pmod 12$.



          Finally $4^8 equiv left(4^2right)^4 equiv 4^4 equiv left(4^2right)^2 equiv 4^2 equiv 4 pmod 12$.



          Then we get the result.



          There are slicker solutions with just a few results from elementary number theory, but this is a very basic method which should be easy enough to follow.






          share|cite|improve this answer






















            up vote
            2
            down vote



            accepted







            up vote
            2
            down vote



            accepted






            First, note that $26 equiv 2 pmod 12$, so $26^32 equiv 2^32 pmod 12$.



            Next, note that $2^4 equiv 16 equiv 4 pmod 12$, so $2^32 equiv left(2^4right)^8 equiv 4 ^8 pmod 12$, and $4^2 equiv 4 pmod 12$.



            Finally $4^8 equiv left(4^2right)^4 equiv 4^4 equiv left(4^2right)^2 equiv 4^2 equiv 4 pmod 12$.



            Then we get the result.



            There are slicker solutions with just a few results from elementary number theory, but this is a very basic method which should be easy enough to follow.






            share|cite|improve this answer












            First, note that $26 equiv 2 pmod 12$, so $26^32 equiv 2^32 pmod 12$.



            Next, note that $2^4 equiv 16 equiv 4 pmod 12$, so $2^32 equiv left(2^4right)^8 equiv 4 ^8 pmod 12$, and $4^2 equiv 4 pmod 12$.



            Finally $4^8 equiv left(4^2right)^4 equiv 4^4 equiv left(4^2right)^2 equiv 4^2 equiv 4 pmod 12$.



            Then we get the result.



            There are slicker solutions with just a few results from elementary number theory, but this is a very basic method which should be easy enough to follow.







            share|cite|improve this answer












            share|cite|improve this answer



            share|cite|improve this answer










            answered 1 hour ago









            Sam Streeter

            840217




            840217




















                up vote
                1
                down vote













                Like



                Get the last two digits of $16^100$ and $17^100$



                How to find last two digits of $2^2016$



                last two digits of $14^5532$?



                Find the last two digits of $2^2156789$



                $$26equiv-1pmod3$$



                $implies26^2nequiv(-1)^2nequiv1pmod3$ where $n$ is any integer



                $implies26^2n+2equiv1cdot26^2pmod3cdot26^2$



                $implies26^2n+2equiv26^2pmod3cdot2^2equiv?$






                share|cite|improve this answer
























                  up vote
                  1
                  down vote













                  Like



                  Get the last two digits of $16^100$ and $17^100$



                  How to find last two digits of $2^2016$



                  last two digits of $14^5532$?



                  Find the last two digits of $2^2156789$



                  $$26equiv-1pmod3$$



                  $implies26^2nequiv(-1)^2nequiv1pmod3$ where $n$ is any integer



                  $implies26^2n+2equiv1cdot26^2pmod3cdot26^2$



                  $implies26^2n+2equiv26^2pmod3cdot2^2equiv?$






                  share|cite|improve this answer






















                    up vote
                    1
                    down vote










                    up vote
                    1
                    down vote









                    Like



                    Get the last two digits of $16^100$ and $17^100$



                    How to find last two digits of $2^2016$



                    last two digits of $14^5532$?



                    Find the last two digits of $2^2156789$



                    $$26equiv-1pmod3$$



                    $implies26^2nequiv(-1)^2nequiv1pmod3$ where $n$ is any integer



                    $implies26^2n+2equiv1cdot26^2pmod3cdot26^2$



                    $implies26^2n+2equiv26^2pmod3cdot2^2equiv?$






                    share|cite|improve this answer












                    Like



                    Get the last two digits of $16^100$ and $17^100$



                    How to find last two digits of $2^2016$



                    last two digits of $14^5532$?



                    Find the last two digits of $2^2156789$



                    $$26equiv-1pmod3$$



                    $implies26^2nequiv(-1)^2nequiv1pmod3$ where $n$ is any integer



                    $implies26^2n+2equiv1cdot26^2pmod3cdot26^2$



                    $implies26^2n+2equiv26^2pmod3cdot2^2equiv?$







                    share|cite|improve this answer












                    share|cite|improve this answer



                    share|cite|improve this answer










                    answered 1 hour ago









                    lab bhattacharjee

                    218k14153268




                    218k14153268




















                        up vote
                        1
                        down vote













                        Note that $26equiv 2pmod12$, so we can as well compute $r=2^32bmod 12$.



                        We have $2^32=12q+r$, so clearly $r=4s$, so we are reduced to compute $2^30bmod 3$ and now we can apply Fermat’s little theorem: $2^2equiv 1pmod 3$; thus $2^30equiv 1=spmod3$ and therefore $r=4$.






                        share|cite|improve this answer
























                          up vote
                          1
                          down vote













                          Note that $26equiv 2pmod12$, so we can as well compute $r=2^32bmod 12$.



                          We have $2^32=12q+r$, so clearly $r=4s$, so we are reduced to compute $2^30bmod 3$ and now we can apply Fermat’s little theorem: $2^2equiv 1pmod 3$; thus $2^30equiv 1=spmod3$ and therefore $r=4$.






                          share|cite|improve this answer






















                            up vote
                            1
                            down vote










                            up vote
                            1
                            down vote









                            Note that $26equiv 2pmod12$, so we can as well compute $r=2^32bmod 12$.



                            We have $2^32=12q+r$, so clearly $r=4s$, so we are reduced to compute $2^30bmod 3$ and now we can apply Fermat’s little theorem: $2^2equiv 1pmod 3$; thus $2^30equiv 1=spmod3$ and therefore $r=4$.






                            share|cite|improve this answer












                            Note that $26equiv 2pmod12$, so we can as well compute $r=2^32bmod 12$.



                            We have $2^32=12q+r$, so clearly $r=4s$, so we are reduced to compute $2^30bmod 3$ and now we can apply Fermat’s little theorem: $2^2equiv 1pmod 3$; thus $2^30equiv 1=spmod3$ and therefore $r=4$.







                            share|cite|improve this answer












                            share|cite|improve this answer



                            share|cite|improve this answer










                            answered 39 mins ago









                            egreg

                            171k1283193




                            171k1283193




















                                up vote
                                0
                                down vote













                                Note that $26^32$ is clearly divisible by $4$ but not by $3$, so the possible congruence classes modulo $12$ are immediately only (represented by) $4$ and $8$. We have $4equiv 1 bmod 3$ and $8equiv 2 equiv -1 bmod 3$, so to distinguish between them we only need to know the answer modulo $3$.



                                This comes down to $(-1)^32=1$ (or we can use little Fermat to the same effect) and the answer is therefore $4$.






                                share|cite|improve this answer




















                                • Splitting the $12$ into the coprime factors $4$ and $3$ is a technique which is substantially generalised in the Chinese Remainder Theorem. The observations in this case are elementary and don't need complex machinery, but it is there if needed to steer the analysis of more complex cases.
                                  – Mark Bennet
                                  14 mins ago














                                up vote
                                0
                                down vote













                                Note that $26^32$ is clearly divisible by $4$ but not by $3$, so the possible congruence classes modulo $12$ are immediately only (represented by) $4$ and $8$. We have $4equiv 1 bmod 3$ and $8equiv 2 equiv -1 bmod 3$, so to distinguish between them we only need to know the answer modulo $3$.



                                This comes down to $(-1)^32=1$ (or we can use little Fermat to the same effect) and the answer is therefore $4$.






                                share|cite|improve this answer




















                                • Splitting the $12$ into the coprime factors $4$ and $3$ is a technique which is substantially generalised in the Chinese Remainder Theorem. The observations in this case are elementary and don't need complex machinery, but it is there if needed to steer the analysis of more complex cases.
                                  – Mark Bennet
                                  14 mins ago












                                up vote
                                0
                                down vote










                                up vote
                                0
                                down vote









                                Note that $26^32$ is clearly divisible by $4$ but not by $3$, so the possible congruence classes modulo $12$ are immediately only (represented by) $4$ and $8$. We have $4equiv 1 bmod 3$ and $8equiv 2 equiv -1 bmod 3$, so to distinguish between them we only need to know the answer modulo $3$.



                                This comes down to $(-1)^32=1$ (or we can use little Fermat to the same effect) and the answer is therefore $4$.






                                share|cite|improve this answer












                                Note that $26^32$ is clearly divisible by $4$ but not by $3$, so the possible congruence classes modulo $12$ are immediately only (represented by) $4$ and $8$. We have $4equiv 1 bmod 3$ and $8equiv 2 equiv -1 bmod 3$, so to distinguish between them we only need to know the answer modulo $3$.



                                This comes down to $(-1)^32=1$ (or we can use little Fermat to the same effect) and the answer is therefore $4$.







                                share|cite|improve this answer












                                share|cite|improve this answer



                                share|cite|improve this answer










                                answered 18 mins ago









                                Mark Bennet

                                79k877177




                                79k877177











                                • Splitting the $12$ into the coprime factors $4$ and $3$ is a technique which is substantially generalised in the Chinese Remainder Theorem. The observations in this case are elementary and don't need complex machinery, but it is there if needed to steer the analysis of more complex cases.
                                  – Mark Bennet
                                  14 mins ago
















                                • Splitting the $12$ into the coprime factors $4$ and $3$ is a technique which is substantially generalised in the Chinese Remainder Theorem. The observations in this case are elementary and don't need complex machinery, but it is there if needed to steer the analysis of more complex cases.
                                  – Mark Bennet
                                  14 mins ago















                                Splitting the $12$ into the coprime factors $4$ and $3$ is a technique which is substantially generalised in the Chinese Remainder Theorem. The observations in this case are elementary and don't need complex machinery, but it is there if needed to steer the analysis of more complex cases.
                                – Mark Bennet
                                14 mins ago




                                Splitting the $12$ into the coprime factors $4$ and $3$ is a technique which is substantially generalised in the Chinese Remainder Theorem. The observations in this case are elementary and don't need complex machinery, but it is there if needed to steer the analysis of more complex cases.
                                – Mark Bennet
                                14 mins ago

















                                 

                                draft saved


                                draft discarded















































                                 


                                draft saved


                                draft discarded














                                StackExchange.ready(
                                function ()
                                StackExchange.openid.initPostLogin('.new-post-login', 'https%3a%2f%2fmath.stackexchange.com%2fquestions%2f2973194%2fcorrect-answer-to-this-modulo%23new-answer', 'question_page');

                                );

                                Post as a guest













































































                                Comments

                                Popular posts from this blog

                                Long meetings (6-7 hours a day): Being “babysat” by supervisor

                                Is the Concept of Multiple Fantasy Races Scientifically Flawed? [closed]

                                Confectionery